LSAT and Law School Admissions Forum

Get expert LSAT preparation and law school admissions advice from PowerScore Test Preparation.

User avatar
 Dave Killoran
PowerScore Staff
  • PowerScore Staff
  • Posts: 5853
  • Joined: Mar 25, 2011
|
#47493
Complete Question Explanation
(The complete setup for this game can be found here: lsat/viewtopic.php?t=17376)

The correct answer choice is (E)

If T sits next to I, then T must sit in chair 4 (and, consequently, R must sit in chair 2). If F sits next to T, then F must sit in chair 3. The only remaining uncertainty involves J and H:
J93_Game_#3_#15_diagram 1.png
In a Not Necessarily True question (or a Could Be True question, for that matter), you should immediately search for an answer that addresses the variable or variables whose placement is uncertain. In this case, J and H are not completely determined, and a scan of the answer choices shows that only answer choice (E) contains J or H. Ultimately, a brief analysis shows that answer choice (E) is the correct answer.
You do not have the required permissions to view the files attached to this post.
 twendell
  • Posts: 8
  • Joined: Aug 05, 2019
|
#71515
Why is F-R-H/J-T-I-S-H/J not allowed? I had two templates that allowed for T-I (this and the one above)
 Jeremy Press
PowerScore Staff
  • PowerScore Staff
  • Posts: 1000
  • Joined: Jun 12, 2017
|
#71528
Hi twendell,

The hypothetical you posed doesn't work here, because of the local condition to the question, which states that "Thelma sits next to Ivan, and ... Frank sits next to Thelma." In your hypothetical, Frank is not sitting next to Thelma. I hope this helps!

Jeremy

Get the most out of your LSAT Prep Plus subscription.

Analyze and track your performance with our Testing and Analytics Package.